Solving a Sequencing Problem: Allocating Cost Savings

Click For Summary
The discussion revolves around a sequencing problem involving three players and six jobs, with specific processing times and cost functions. The user has calculated an optimal job order and is seeking to allocate cost savings based on this new sequence. They have computed values for g_ij, which represent the cost differences between job pairs. However, there is confusion regarding the clarity of the problem statement, particularly in defining variables and the objective of the cost savings allocation. The user is encouraged to clarify their definitions and objectives for better assistance.
Cyn
Messages
7
Reaction score
0
<Moderator's note: LaTeX code edited.>

1. Homework Statement

Hi, I have a question.
I have a sequencing problem ##(N, \sigma_{0}##, ##\{p_{j}\}_{j\in J}, \{c_{i}\}_{i\in N})## with ##N = \{1,2,3\}## the set of players, and ##J = \{A,B,C,D,E,F\}## the set of jobs. The processing times of the jobs are:
$$p_{A} =8\\
p_{B} =10 \\
p_{C} = 15\\
p_{D} = 5 \\
p_{E}=6\\
p_{F}= 12$$
De set jobs van de spelers, ##J(i)##, zijn:
$$J(1) = \{B,E\}, J(2) = \{A,D\}, J(3) =\{C,F\},$$
##c_{1}(\sigma) = 7min\{C_{B}(\sigma),C_{E}(\sigma)\}##,
##c_{2}(\sigma) = 4min\{C_{A}(\sigma),C_{D}(\sigma)\}##,
##c_{3}(\sigma) = 12min\{C_{C}(\sigma),C_{F} (\sigma)\}##,
##\sigma_{0} = (A B C D E F).##

Homework Equations


What is the allocation of the cost savings?

The Attempt at a Solution


I have calculated the $\alpha$'s and the optimal order begins with (EFD). Now I want to calculate an allocation of the cost savings.
I have said that my new order becomes ##(EFDCBA)##. So ##MP(\sigma_{0})=\{(A,B), (A,C), (A,D), (A,E), (A,F), (B,C), (B,D), (B,E), (B,F), (C,D), (C,E), (C,F), (D, E), (D,F)\}##
Then, I have calcultated ##g_{ij}. (g_{ij} = \alpha_{j}p_{i}-\alpha_{i}p_{j}).##

##g_{AB} = 16;
g_{AE} = 32;
g_{BC} = 15;
g_{BD} = 5;
g_{BE} = 28;
g_{BF} = 36;
g_{CE} = 33;
g_{DE} = 11##
If we use EGS, is it correct that ##EGS_{1}= 102##?​
 
Last edited by a moderator:
Physics news on Phys.org
Cyn said:

Homework Statement


Hi, I have a question.
I have a sequencing problem (N, $\sigma_{0}$, $\{p_{j}\}_{j\in J}, \{c_{i}\}_{i\in N}$) with N = \{1,2,3\} the set of players, and J = \{A,B,C,D,E,F\} the set of jobs. The processing times of the jobs are:
$$p_{A} =8\\
p_{B} =10 \\
p_{C} = 15\\
p_{D} = 5 \\
p_{E}=6\\
p_{F}= 12$$
De set jobs van de spelers, $J(i)$, zijn:
$$J(1) = \{B,E\}, J(2) = \{A,D\}, J(3) =\{C,F\},$$
$c_{1}(\sigma) = 7min\{C_{B}(\sigma),C_{E}(\sigma)\}$,
$c_{2}(\sigma) = 4min\{C_{A}(\sigma),C_{D}(\sigma)\}$,
$c_{3}(\sigma) = 12min\{C_{C}(\sigma),C_{F} (\sigma)\}$,
$\sigma_{0}$ = (A B C D E F).

Homework Equations


What is the allocation of the cost savings?

The Attempt at a Solution


I have calculated the $\alpha$'s and the optimal order begins with (EFD). Now I want to calculate an allocation of the cost savings.
I have said that my new order becomes (EFDCBA). So MP($\sigma_{0}$)={(A,B), (A,C), (A,D), (A,E), (A,F), (B,C), (B,D), (B,E), (B,F), (C,D), (C,E), (C,F), (D, E), (D,F)}
Then, I have calcultated $g_{ij}$. ($g_{ij} = \alpha_{j}p_{i}-\alpha_{i}p_{j}$).

$g_{AB} = 16;
g_{AE} = 32;
g_{BC} = 15;
g_{BD} = 5;
g_{BE} = 28;
g_{BF} = 36;
g_{CE} = 33;
g_{DE} = 11$
If we use EGS, is it correct that $EGS_{1}= 102?​
Your problem statement is very difficult to understand.
1) One thing that would help a lot would be to use "##" instead of "$" to delimit your LaTeX notation. Then it would display correctly.
2) Please give definitions of the variables. What do p, c, g, σ and α represent?
3) You give as your goal that you want to allocate cost savings, but you have not clearly defined your objective in mathematical terms. Cost savings relative to what? What are you optimizing?
 
Question: A clock's minute hand has length 4 and its hour hand has length 3. What is the distance between the tips at the moment when it is increasing most rapidly?(Putnam Exam Question) Answer: Making assumption that both the hands moves at constant angular velocities, the answer is ## \sqrt{7} .## But don't you think this assumption is somewhat doubtful and wrong?

Similar threads

  • · Replies 71 ·
3
Replies
71
Views
2K
  • · Replies 17 ·
Replies
17
Views
2K
  • · Replies 1 ·
Replies
1
Views
1K
  • · Replies 1 ·
Replies
1
Views
1K
  • · Replies 16 ·
Replies
16
Views
4K
  • · Replies 1 ·
Replies
1
Views
2K
  • · Replies 9 ·
Replies
9
Views
2K
  • · Replies 9 ·
Replies
9
Views
3K
  • · Replies 6 ·
Replies
6
Views
2K
  • · Replies 1 ·
Replies
1
Views
2K